K
Khách

Hãy nhập câu hỏi của bạn vào đây, nếu là tài khoản VIP, bạn sẽ được ưu tiên trả lời.

7 tháng 10 2021

Mình không biết nha tạm thời bạn hỏi bạn khác đi 😅

NV
14 tháng 3 2022

- Với \(x=1\) ko thỏa mãn

- Với \(x=2\Rightarrow\dfrac{2}{2y+2}\in Z\Rightarrow\dfrac{1}{y+1}\in Z\Rightarrow y=\left\{-2;0\right\}\) ko thỏa mãn

- Với \(x\ge3\)

\(x^2-2⋮xy+2\Rightarrow x\left(xy+2\right)-y\left(x^2-2\right)⋮xy+2\)

\(\Rightarrow2\left(x+y\right)⋮xy+2\)

\(\Rightarrow\left(x-2\right)\left(y-2\right)\le2\)

\(\Rightarrow y-2\le\dfrac{2}{x-2}\le\dfrac{2}{3-2}=2\Rightarrow y\le4\)

\(\Rightarrow y=\left\{1;2;3;4\right\}\)

Lần lượt thay 3 giá trị của y vào pt biểu thức ban đầu

Ví dụ: \(y=1\Rightarrow\dfrac{x^2-2}{x+2}\in Z\Rightarrow x-2+\dfrac{2}{x+2}\in Z\)

\(\Rightarrow x+2=Ư\left(2\right)\Rightarrow\) ko tồn tại x nguyên dương t/m

Tương tự...

14 tháng 3 2022

Em cả mơn thầy 

Thầy mãi đỉnh

9 tháng 10 2018

Giả sử (x;y) là cặp số nguyên dương cần tìm. Khi đó ta có: 
(xy-1) I (x^3+x) => (xy-1) I x.(x^2+1) (1) 
Do (x; xy-1) =1 ( Thật vậy: gọi (x;xy-1) =d => d I x => d I xy => d I 1). 
Nên từ (1) ta có: 
(xy-1) I (x^2+1) 
=> (xy-1) I (x^2+1+xy -1) => (xy-1) I (x^2+xy) => (xy-1) I x.(x+y) => (xy-1) I (x+y) 
Điều đó có nghĩa là tồn tại z ∈ N* sao cho: 
x+y = z(xy-1) <=> x+y+z =xyz (2) 

[Đây lại có vẻ là 1 bài toán khác] 
Do vai trò bình đẳng nên ta giả sử: x ≥ y ≥ z. 
Từ (2) ta có: x+y+z ≤ 3x => 3x ≥ xyz => 3 ≥ yz ≥ z^2 => z=1 
=> 3 ≥ y => y ∈ {1;2;3} 
Nếu y=1: x+2 =x (loại) 
Nếu y=2: (2) trở thành x+3 =2x => x=3 
Nếu y=3: x+4 = 3x => x=2 (loại vì ta có x≥y) 
Vậy khi x ≥ y ≥ z thì (2) có 1 nghiệm (x;y;z) là (3;2;1) 
Hoán vị vòng quanh được 6 nghiệm là: .....[bạn tự viết nhé] 

Vậy bài toán đã cho có 6 nghiệm (x;y) là : .... [viết y chang nhưng bỏ z đi]

9 tháng 10 2018

 Giả sử (x;y) là cặp số nguyên dương cần tìm. Khi đó ta có: 
(xy-1) I (x^3+x) => (xy-1) I x.(x^2+1) (1) 
Do (x; xy-1) =1 ( Thật vậy: gọi (x;xy-1) =d => d I x => d I xy => d I 1).
Nên từ (1) ta có: 
(xy-1) I (x^2+1) 
=> (xy-1) I (x^2+1+xy -1) => (xy-1) I (x^2+xy) => (xy-1) I x.(x+y) => (xy-1) I (x+y) 
Điều đó có nghĩa là tồn tại z ∈ N* sao cho: 
x+y = z(xy-1) <=> x+y+z =xyz (2) 

[Đây lại có vẻ là 1 bài toán khác] 
Do vai trò bình đẳng nên ta giả sử: x ≥ y ≥ z. 
Từ (2) ta có: x+y+z ≤ 3x => 3x ≥ xyz => 3 ≥ yz ≥ z^2 => z=1 
=> 3 ≥ y => y ∈ {1;2;3} 
Nếu y=1: x+2 =x (loại) 
Nếu y=2: (2) trở thành x+3 =2x => x=3 
Nếu y=3: x+4 = 3x => x=2 (loại vì ta có x≥y) 
Vậy khi x ≥ y ≥ z thì (2) có 1 nghiệm (x;y;z) là (3;2;1) 
Hoán vị vòng quanh được 6 nghiệm là: .....[bạn tự viết nhé] 

Vậy bài toán đã cho có 6 nghiệm (x;y) là : .... [viết y chang nhưng bỏ z đi]

27 tháng 3 2020

Bài 1 : 

Phương trình <=> 2x . x2 = ( 3y + 1 ) + 15

Vì \(\hept{\begin{cases}3y+1\equiv1\left(mod3\right)\\15\equiv0\left(mod3\right)\end{cases}\Rightarrow\left(3y+1\right)^2+15\equiv1\left(mod3\right)}\)

\(\Rightarrow2^x.x^2\equiv1\left(mod3\right)\Rightarrow x^2\equiv1\left(mod3\right)\)

( Vì số  chính phương chia 3 dư 0 hoặc 1 ) 

\(\Rightarrow2^x\equiv1\left(mod3\right)\Rightarrow x\equiv2k\left(k\inℕ\right)\)

Vậy \(2^{2k}.\left(2k\right)^2-\left(3y+1\right)^2=15\Leftrightarrow\left(2^k.2.k-3y-1\right).\left(2^k.2k+3y+1\right)=15\)

Vì y ,k \(\inℕ\)nên 2k . 2k + 3y + 1 > 2k .2k - 3y-1>0

Vậy ta có các trường hợp: 

\(+\hept{\begin{cases}2k.2k-3y-1=1\\2k.2k+3y+1=15\end{cases}\Leftrightarrow\hept{\begin{cases}2k.2k=8\\3y+1=7\end{cases}\Rightarrow}k\notinℕ\left(L\right)}\)

\(+,\hept{\begin{cases}2k.2k-3y-1=3\\2k.2k+3y+1=5\end{cases}\Leftrightarrow\hept{\begin{cases}2k.2k=4\\3y+1=1\end{cases}\Rightarrow}\hept{\begin{cases}k=1\\y=0\end{cases}\left(TM\right)}}\)

Vậy ( x ; y ) =( 2 ; 0 ) 

27 tháng 3 2020

Bài 3: 

Giả sử \(5^p-2^p=a^m\)    \(\left(a;m\inℕ,a,m\ge2\right)\)

Với \(p=2\Rightarrow a^m=21\left(l\right)\)

Với \(p=3\Rightarrow a^m=117\left(l\right)\)

Với \(p>3\)nên p lẻ, ta có

\(5^p-2^p=3\left(5^{p-1}+2.5^{p-2}+...+2^{p-1}\right)\Rightarrow5^p-2^p=3^k\left(1\right)\)    \(\left(k\inℕ,k\ge2\right)\)

Mà \(5\equiv2\left(mod3\right)\Rightarrow5^x.2^{p-1-x}\equiv2^{p-1}\left(mod3\right),x=\overline{1,p-1}\)

\(\Rightarrow5^{p-1}+2.5^{p-2}+...+2^{p-1}\equiv p.2^{p-1}\left(mod3\right)\)

Vì p và \(2^{p-1}\)không chia hết cho 3 nên \(5^{p-1}+2.5^{p-2}+...+2^{p-1}⋮̸3\)

Do đó: \(5^p-2^p\ne3^k\), mâu thuẫn với (1). Suy ra giả sử là điều vô lý

\(\rightarrowĐPCM\)

9 tháng 10 2018

can you hẹp me?? mk đang cần gấp 

24 tháng 12 2019

Vì gcd(x,x2+1)=1gcd(x,x2+1)=1 suy ra
Hoặc xy−1|;xxy−1|;x hoặc xy−1|x2+1xy−1|x2+1
Trường hợp 1 ta có: {x−1≤xy−1≤xxy−1|x}⇒[xy−1=xxy−1=1]⇒[x(y−1)=1xy=2]⇒[x=1;y=2x=2;y=1]{x−1≤xy−1≤xxy−1|x}⇒[xy−1=xxy−1=1]⇒[x(y−1)=1xy=2]⇒[x=1;y=2x=2;y=1]

Trường hợp 2 xét modulo xx ta có: {xy−1≡−1(modx)x2+1≡1(modx)}⇒−1≡1(modx)⇒2≡0(modx)⇒x=1 hoặc x=2{xy−1≡−1(modx)x2+1≡1(modx)}⇒−1≡1(modx)⇒2≡0(modx)⇒x=1 hoặc x=2

Thay các giá trị xx vào biểu thức ta tìm được yy

Cuối cùng các giá trị phải tìm là (x,y)∈{(1,2);(1,3);(2,1);(2,3)}(x,y)∈{(1,2);(1,3);(2,1);(2,3)}

k mik nha